Next: 4.89(a), §2 Solution Up: 4.89(a) Previous: 4.89(a)

4.89(a), §1 Asked

Asked: Are (1,2,-3,1), (3,7,1,-2), and (1,3,7,-4) independent?


Next: 4.89(a), §2 Solution Up: 4.89(a) Previous: 4.89(a)